¿Por qué tomar trabajo de un sistema no reduce su entropía?

En el libro Thermal Physics de Daniel Schroeder, señala que a medida que se pone energía en un sistema, su entropía aumenta (hay algunas excepciones a esto en general, pero para esta pregunta podemos ignorar esto). Sin embargo, a medida que se introduce energía en el sistema, la entropía sigue aumentando, pero en cantidades cada vez menores. También definimos el recíproco del cambio de Entropía sobre el cambio de Energía como la Temperatura del sistema:

1 T = S tu

Todo esto tiene sentido: un objeto de temperatura más baja es aquel en el que se gana mucha entropía si se le agrega energía, y un objeto de temperatura más alta es aquel en el que se pierde muy poca entropía si se le quita energía. A partir de esto, es fácil ver por qué el objeto de temperatura más alta dará energía espontáneamente al objeto de temperatura más baja si se ponen en contacto térmico entre sí.

Esto también tiene sentido desde la interpretación de la entropía como el número de "microestados" del sistema. Claro, eliminar energía hará que el microestado del objeto de temperatura más alta sea más seguro (porque ahora hay menos), pero hacer que el objeto de temperatura más baja reciba esa energía agregará mucha más incertidumbre al microestado, más que compensar los microestados perdidos. en el sistema del objeto de mayor temperatura.

Pero si todo esto fuera cierto, entonces no debería importar cómo entra/sale la energía de mi sistema. Es decir, no debería importar si la energía entra o sale a través del flujo de calor o si se realizó trabajo sobre ella. Ciertamente, podemos ver en la fórmula anterior para la temperatura que la temperatura depende de cómo la entropía S cambia cuando hay un cambio en la energía del sistema tu . No hay referencia a q , el calor puesto en el sistema ya sea en la fórmula o en la interpretación de la entropía anterior.

Por eso me confunde cuando habla de motores térmicos. Schroeder trata de determinar la eficiencia máxima de un motor térmico que toma calor q h de un depósito caliente, apaga el trabajo W y arroja calor q C como desperdicio Comenzando con la definición de eficiencia mi = W / q h él pone en dos restricciones. La primera restricción proviene de la Primera Ley de la Termodinámica y tiene sentido:

q h = q C + W

La segunda restricción proviene de la Segunda Ley y me tiene confundido:

q C T C q h T h

¿Por qué no hay un término para la entropía que se lleva a cabo cuando se realiza trabajo en el sistema? En otras palabras, ¿por qué la restricción de la segunda ley no es así?

q C T C + W T ? q h T h

(Supongo T ? sería la temperatura del motor, entre T C y T h , cuando está haciendo el trabajo.)

Sin embargo, la restricción no está escrita de esta manera, entonces, ¿por qué tomar trabajo de un sistema no reduce su entropía?

T ? =
Sin embargo, hemos dicho que eso T ? debe estar entre T C y T h ... @hyportnex
de hecho lo has dicho pero eso esta mal
Sé que algo está mal (o eso o acabo de captar algo que todos los que alguna vez tomaron el tema se perdieron) ... Solo estoy tratando de entender la razón por la cual está mal. De todos modos, debido a que se encuentra entre los dos depósitos de calor, la temperatura de la máquina siempre debe estar entre T C y T h . ¿Estás de acuerdo con esto, o es esta la parte que está mal? @hyportnex
En su ejemplo, hay tres fuentes de energía (sumideros): dos depósitos de calor y un depósito de trabajo. Sus temperaturas, cualquiera que sea el significado del término "temperatura" para una fuente de energía, son independientes entre sí; no hay un orden natural de esas temperaturas a menos que haya un flujo de calor (entropía y energía) de una a otra sin que se realice trabajo. Dado que un depósito de trabajo transfiere energía y entropía cero (proceso reversible), su temperatura se puede tomar como .

Respuestas (1)

No TODOS los cambios en la energía interna del sistema corresponden a un cambio en su entropía. Esto es importante para este tema ya que el trabajo es precisamente el cambio de energía que no corresponde a un cambio de entropía.

La fórmula anterior para la temperatura es algo engañosa ya que no se nota lo que se mantiene constante durante la derivada parcial. Es muy importante que todas las variables externas, como el volumen, se mantengan constantes. Eso limita el cambio de energía para que sea solo el calor, que es lo que hace d S = d q / T en lugar de cualquier viejo d tu / T . Un cambio de volumen dará como resultado un cambio de energía que no está relacionado con el cambio de entropía, es decir, el trabajo.

La segunda ley explica solo el cambio de entropía, no el cambio total de energía. Entonces agregando un + W / T El término es inapropiado: eso introduciría un término que está rastreando algo no relacionado con el cambio de entropía, el trabajo.

Gracias por su respuesta... Usted afirma que "un cambio de volumen dará como resultado un cambio de energía que no está relacionado con el cambio de entropía", sin embargo, comprimir un gas debería elevar su temperatura, y toda esa energía no debería significar que hay más incertidumbre del estado del gas (por lo tanto, aumento de la entropía)? Además, ¿la identidad termodinámica no d S = PAG / T d V (todo menos el volumen se mantiene constante) ¿muestra que cambiar el volumen de algo puede aumentar su entropía? @Profesor de física
No es lo mismo temperatura que entropía. Considere un ciclo de Otto ( en.wikipedia.org/wiki/Otto_cycle ). Tiene dos tiempos de compresión/expansión que son isoentrópicos: la entropía no cambia, aunque sí el volumen y la temperatura. Tenga en cuenta que el cambio en el volumen PUEDE resultar en un aumento de la entropía, especialmente si es rápido. La identidad termodinámica que declaras se mantiene solo bajo energía interna constante. Generalmente d S = 1 T ( PAG d V + d tu ) . La compresión lenta resultará en d tu = PAG d V de modo que d S = 0 .
Guau, entonces la respuesta es que cuando se realiza trabajo en un sistema a través de la compresión, la entropía adicional ganada por toda esa energía adicional (más incertidumbre de velocidad) en el sistema se cancela exactamente por el espacio más pequeño (menos incertidumbre de posición). Realmente es sorprendente que los dos procesos en competencia se anulen exactamente entre sí, ya que el aumento/disminución de la entropía ocurre a través de dos procesos aparentemente no relacionados: uno es un aumento en la incertidumbre de la velocidad y el otro es una disminución en la incertidumbre de la posición. Tampoco importa la forma del recipiente. @Profesor de física
Tenga en cuenta que esto es una idealización: es cierto para un proceso cuasiestático, pero en la práctica tendrá una tasa finita e irreversibilidad y acumulación de entropía hasta cierto punto. Además, no es correcto para algunos casos, consulte el 'teorema adiabático cuántico' y el 'paso a nivel'.